Difference between revisions of "1996 AHSME Problems/Problem 29"
(→Solution 2) |
|||
(5 intermediate revisions by 2 users not shown) | |||
Line 37: | Line 37: | ||
In this case, <math>6n = 3^4\cdot 2^6</math>, which has <math>5\cdot 7 = 35</math> factors, and the answer is <math>\boxed{C}</math> | In this case, <math>6n = 3^4\cdot 2^6</math>, which has <math>5\cdot 7 = 35</math> factors, and the answer is <math>\boxed{C}</math> | ||
+ | |||
+ | == Solution 2 == | ||
+ | Because <math>2n</math> has <math>28</math> factors and <math>3n</math> has <math>30</math> factors, we should rewrite the number <math>n = 2^{e_1}3^{e_2}... p_n^{e_n}</math> | ||
+ | As the formula for the number of divisors for such a number gives: | ||
+ | <math>(e_1+1)(e_2+1)... (e_n+1)</math> | ||
+ | We plug in the variations we need to make for the cases <math>2n</math> and <math>3n</math>. | ||
+ | <math>2n</math> has <math>(e_1+2)(e_2+1)(e_3+1)... (e_n+1) = 28</math> | ||
+ | <math>3n</math> has <math>(e_1+1)(e_2+2)(e_3+1)...(e_n+1) = 30</math> | ||
+ | |||
+ | If we take the top and divide by the bottom, we get the following equation: | ||
+ | <math>\frac{(e_1+2)(e_2+1)}{(e_1+1)(e_2+2)} = \frac{14}{15}</math>. Letting <math>e_1=x</math> and <math>e_2 = y</math> for convenience and expanding this out gives us: | ||
+ | <math>xy-13x+16y+2=0</math> | ||
+ | |||
+ | We can use Simon's Favorite Factoring Trick (SFFT) to turn this back into: | ||
+ | <math>(x+16)(y-13) +2 + 208 = 0</math> or | ||
+ | <math>(x+16)(y-13) = - 210</math> | ||
+ | |||
+ | As we want to be dealing with rather reasonable numbers for <math>x</math> and <math>y</math>, we try to make the <math>x+16</math> term the slightly larger term and the <math>y-13</math> term the slightly smaller term. This effect is achieved when <math>x+16 = 21</math> and <math>y-13 = -10</math>. Therefore, <math>x = 5, y = 3</math>. We get that this already satisfies the requirements for the number we are looking for, and we take <math>(5+2)(3+2) = \boxed{35}</math> | ||
+ | |||
+ | ==Solution 3 (Alcumus Solution)== | ||
+ | Let <math>\, 2^{e_1} 3^{e_2} 5^{e_3} \cdots \,</math> be the prime factorization of <math>\, n</math>. Then the number of positive divisors of <math>\, n \,</math> is <math>\, (e_1 + 1)(e_2 + 1)(e_3 + 1) \cdots \; </math>. In view of the given information, we have <cmath> 28 = (e_1 + 2)(e_2 + 1)P </cmath>and <cmath> 30 = (e_1 + 1)(e_2 + 2)P, </cmath>where <math>\, P = (e_3 + 1)(e_4 + 1) \cdots \; </math>. Subtracting the first equation from the second, we obtain <math>\, 2 = (e_1 - e_2)P, \,</math> so either <math>\, e_1 - e_2 = 1 \,</math> and <math>\, P = 2, \,</math> or <math>\, e_1 - e_2 = 2 \,</math> and <math>\, P = 1</math>. The first case yields <math>\, 14 = (e_1 + 2)e_1 \,</math> and <math>\, (e_1 + 1)^2 = 15</math>; since <math>\, e_1 \,</math> is a nonnegative integer, this is impossible. In the second case, <math>\, e_2 = e_1 - 2 \,</math> and <math>\, 30 = (e_1 + 1)e_1, \,</math> from which we find <math>\, e_1 = 5 \,</math> and <math>\, e_2 = 3</math>. Thus <math>\, n = 2^5 3^3, \,</math> so <math>\, 6n = 2^6 3^4 \,</math> has <math>\, (6+1)(4+1) = \boxed{35} \,</math> positive divisors. | ||
==See also== | ==See also== | ||
{{AHSME box|year=1996|num-b=28|num-a=30}} | {{AHSME box|year=1996|num-b=28|num-a=30}} | ||
{{MAA Notice}} | {{MAA Notice}} |
Latest revision as of 04:52, 26 October 2023
Problem
If is a positive integer such that has positive divisors and has positive divisors, then how many positive divisors does have?
Solution 1
Working with the second part of the problem first, we know that has divisors. We try to find the various possible prime factorizations of by splitting into various products of or integers.
The variables are different prime factors, and one of them must be . We now try to count the factors of , to see which prime factorization is correct and has factors.
In the first case, is the only possibility. This gives , which has factors, which is way too many.
In the second case, gives . If , then there are factors, while if , there are factors.
In the second case, gives . If , then there are factors, while if , there are factors.
In the third case, gives . If , then there are factors, while if , there are factors.
In the third case, gives . If , then there are factors, while if , there are factors.
In the fourth case, gives . If , then there are factors. This is the factorization we want.
Thus, , which has factors, and , which has factors.
In this case, , which has factors, and the answer is
Solution 2
Because has factors and has factors, we should rewrite the number As the formula for the number of divisors for such a number gives: We plug in the variations we need to make for the cases and . has has
If we take the top and divide by the bottom, we get the following equation: . Letting and for convenience and expanding this out gives us:
We can use Simon's Favorite Factoring Trick (SFFT) to turn this back into: or
As we want to be dealing with rather reasonable numbers for and , we try to make the term the slightly larger term and the term the slightly smaller term. This effect is achieved when and . Therefore, . We get that this already satisfies the requirements for the number we are looking for, and we take
Solution 3 (Alcumus Solution)
Let be the prime factorization of . Then the number of positive divisors of is . In view of the given information, we have and where . Subtracting the first equation from the second, we obtain so either and or and . The first case yields and ; since is a nonnegative integer, this is impossible. In the second case, and from which we find and . Thus so has positive divisors.
See also
1996 AHSME (Problems • Answer Key • Resources) | ||
Preceded by Problem 28 |
Followed by Problem 30 | |
1 • 2 • 3 • 4 • 5 • 6 • 7 • 8 • 9 • 10 • 11 • 12 • 13 • 14 • 15 • 16 • 17 • 18 • 19 • 20 • 21 • 22 • 23 • 24 • 25 • 26 • 27 • 28 • 29 • 30 | ||
All AHSME Problems and Solutions |
The problems on this page are copyrighted by the Mathematical Association of America's American Mathematics Competitions.